Thomas' Calculus 13th Edition

Published by Pearson
ISBN 10: 0-32187-896-5
ISBN 13: 978-0-32187-896-0

Chapter 5: Integrals - Section 5.2 - Sigma Notation and Limits of Finite Sums - Exercises 5.2 - Page 266: 43

Answer

$\dfrac{5}{6}$

Work Step by Step

Consider $f(x)=x+x^2$ This implies that $\Sigma_{i=1}^n (\dfrac{1}{n}) (k_i+k_i^2)=(\dfrac{1}{n}) \Sigma_{i=1}^n (\dfrac{i}{n}+\dfrac{i^2}{n^2})$ $(\dfrac{1}{n^3})\Sigma_{i=1}^n i+(\dfrac{1}{n^3})\Sigma_{i=1}^n i^2=\dfrac{5n^3+6n^2+n}{6n^3}=\dfrac{5+\dfrac{6}{n}+\dfrac{1}{n^2}}{6}$ Now, $\lim\limits_{n \to \infty}\dfrac{5+\dfrac{6}{n}+\dfrac{1}{n^2}}{6}=\dfrac{5}{6}$
Update this answer!

You can help us out by revising, improving and updating this answer.

Update this answer

After you claim an answer you’ll have 24 hours to send in a draft. An editor will review the submission and either publish your submission or provide feedback.